Sunteți pe pagina 1din 92

A 19 year old female is admitted with suspected meningitis.

The House
Officer is due to perform a
lumbar puncture. What is the most likely structure first encountered
when the needle is inserted?
A. Ligamentum flavum
B. Denticulate ligament
C. Dural sheath
D. Pia Mater
E. Supraspinous ligament
Theme from September 2012 exam

A 34 year old male is being examined in the pre-operative assessment


clinic. A murmur is identified in
the 5th intercostal space just next to the left side of the sternum. From
where is it most likely to have
originated?
A. Mitral valve
B. Aortic valve
C. Pulmonary valve
D. Right ventricular aneurysm
E. Tricuspid valve
Theme from September 2012 Exam

What is the correct embryological origin of the stapes?


A. First pharyngeal arch
B. Second pharyngeal arch
C. Third pharyngeal arch
D. Fourth pharyngeal arch
E. Fifth pharyngeal arch
Embryological origin stapes = 2nd
pharyngeal arch
Theme from April 2012 Exam

Which muscle initiates abduction of the shoulder?


A. Infraspinatus
B. Latissimus dorsi
C. Supraspinatus
D. Deltoid
E. Teres major
Theme from September 2012 exam

What is the most important structure involved in supporting the


uterus?
A. Round ligament
B. Broad ligament
C. Uterosacral ligaments
D. Cardinal ligaments
E. Central perineal tendon
Theme from September 2012 Exam

78 year old man is due to undergo an endarterectomy of the internal


carotid artery. Which of the
following nervous structures are most at risk during the dissection?
A. Recurrent laryngeal nerve
B. Sympathetic chain
C. Hypoglossal nerve
D. Phrenic nerve
E. Lingual nerve
Nerves at risk during a carotid
endarterectomy:
Hypoglossal nerve
Greater auricular nerve
Superior laryngeal nerve
Theme from May 2011 exam
Theme from January 2013 Exam
Which of the following structures is not transmitted by the jugular
foramen?
A. Hypoglossal nerve
B. Accessory nerve
C. Internal jugular vein
D. Inferior petrosal sinus
E. Vagus nerve
Contents of the jugular foramen:
Anterior: inferior petrosal sinus
Intermediate: glossopharyngeal, vagus, and accessory nerves
Posterior: sigmoid sinus (becoming the internal jugular vein) and some
meningeal branches from the occipital
and ascending pharyngeal arteries
Theme from 2009 exam

With regards to the jugular vein, which of the following statements is


untrue?
A. It lies within the carotid sheath
B. It is the continuation of the sigmoid sinus
C. The terminal part of the thoracic duct crosses anterior to it to insert
into the right subclavian vein
D. The hypoglossal nerve is closely related to it as it passes near the
atlas
E. The vagus nerve is closely related to it within the carotid sheath
Theme from April 2013 Exam

A 28 year old man requires a urethral catheter to be inserted prior to


undergoing a splenectomy. Where is the
first site of resistance to be encountered on inserting the catheter?
A. Bulbar urethra
B. Membranous urethra
C. Internal sphincter
D. Prostatic urethra
E. Bladder neck
Theme from 2011 exam
Theme from January 2012 Exam
A 23 year old man undergoes an orchidectomy. The right testicular vein
is ligated; into which structure does it
drain?
A. Right renal vein
B. Inferior vena cava
C. Common iliac vein
D. Internal iliac vein
E. External iliac vein
Theme from April 2012 Exam

A 24 year old man falls and sustains a fracture through his scaphoid
bone. From which of the following areas
does the scaphoid derive the majority of its blood supply?
A. From its proximal medial border
B. From its proximal lateral border
C. From its proximal posterior surface
D. From the proximal end
E. From the distal end
Theme from April 2012 Exam

Which of the following forms the medial wall of the femoral canal?
A. Pectineal ligament
B. Adductor longus
C. Sartorius
D. Lacunar ligament
E. Inguinal ligament
Theme from September 2011 Exam
Theme from September 2012 Exam

A 67 year old man is undergoing a transurethral resection of a bladder


tumour using diathermy. Suddenly during the
procedure the patients leg begins to twitch. Stimulation of which of the
following nerves is the most likely cause?
A. Femoral
B. Pudendal
C. Sciatic
D. Obturator
E. Gluteal
Theme from January 2011 Exam
Theme from January 2013 Exam

A 5 year old boy is playing with some small ball bearings. Unfortunately
he inhales one. To which of the following lung
regions is the ball most likely to settle?
A. Right lower lobe
B. Left main bronchus
C. Right upper lobe
D. Left lower lobe
E. None of the above
Theme from September 2011 Exam
Theme from January 2013 Exam

An 18 year old man is cutting some plants when a small piece of


vegetable matter enters his eye. His eye
becomes watery. Which of the following is responsible for relaying
parasympathetic neuronal signals to the
lacrimal apparatus?
A. Pterygopalatine ganglion
B. Otic ganglion
C. Submandibular ganglion
D. Ciliary ganglion
E. None of the above
Theme from January 2013 Exam

A 43 year old lady is undergoing a total thyroidectomy for an extremely


large goitre. The surgeons decide that
access may be improved by division of the infra hyoid strap muscles. At
which of the following sites should
they be divided?
A. In their upper half
B. In their lower half
C. In the middle
D. At their origin from the hyoid
E. At the point of their insertion
Theme from 2009 Exam

A 22 year old women has recently undergone a surgical excision of the


submandibular gland. She presents to the follow
up clinic with a complaint of tongue weakness on the ipsilateral side to
her surgery. Which nerve has been damaged?
A. Hypoglossal nerve
B. Lingual nerve
C. Inferior alveolar nerve
D. Facial nerve
E. Lesser petrosal nerve
Three cranial nerves may be injured during
submandibular gland excision.
Marginal mandibular branch of the facial nerve
Lingual nerve
Hypoglossal nerve
Theme from April 2012 Exam

You decide to take an arterial blood gas from the femoral artery. Where
should the needle be inserted to gain the sample?
A. Mid point of the inguinal ligament
B. Mid inguinal point
C. 2cm inferomedially to the pubic tubercle
D. 2cm superomedially to the pubic tubercle
E. 3cm inferolaterally to the deep inguinal ring
Theme from September 2012 Exam

A 67 year old man undergoes a carotid endarterectomy and seems to


recover well following surgery. When he is reviewed
on the ward post operatively he complains that his voice is hoarse. What
is the most likely cause?
A. Damage to the accessory nerve
B. Damage to the cervical plexus
C. Damage to the glossopharyngeal nerve
D. Damage to the hypoglossal nerve
E. Damage to the vagus
Theme from April 2013 Exam

A 25 year old man has an inguinal hernia, which of the following


structures must be divided (at open surgery)
to gain access to the inguinal canal?
A. Transversalis fascia
B. External oblique aponeurosis
C. Conjoint tendon
D. Rectus abdominis
E. Inferior epigastric artery
Theme from January 2013 Exam

A 34 year old man is shot in the postero- inferior aspect of his thigh.
Which of the following lies at the most lateral aspect
of the popliteal fossa?
A. Popliteal artery
B. Popliteal vein
C. Common peroneal nerve
D. Tibial nerve
E. Small saphenous vein
Theme from April 2012 Exam

A 67 year old man has an abdominal aortic aneurysm which displaces


the left renal vein. Which branch of the
aorta is most likely to affected at this level?
A. Inferior mesenteric artery
B. Superior mesenteric artery
C. Coeliac axis
D. Testicular artery
E. None of the above
Theme from April 2013 Exam
A 12 year old boy undergoes surgery for recurrent mastoid infections.
Post operatively he complains of an altered taste
sensation. Which of the following nerves has been injured?
A. Glossopharyngeal
B. Greater petrosal
C. Olfactory
D. Trigeminal
E. Chorda tympani
Theme from April 2012 Exam

What is the anatomical level of the transpyloric plane?


A. T11
B. T12
C. L1
D. L4
E. T10
Theme from September 2012 Exam
Theme from January 2013 Exam

A 35 year old man falls and sustains a fracture to the middle third of his
clavicle. Which vessel is at greatest risk
of injury?
A. Subclavian vein
B. Subclavian artery
C. External carotid artery
D. Internal carotid artery
E. Vertebral artery
Theme from April 2013 Exam

A 33 year old man sustains an injury to his forearm and wrist. When
examined in clinic he is unable to adduct
his thumb. What is the most likely underlying nerve lesion?
A. Radial nerve
B. Superficial branch of the ulnar nerve
C. Median nerve
D. Posterior interosseous nerve
E. Deep branch of the ulnar nerve
Theme from April 2013 Exam

A 63 year old man who smokes heavily presents with dyspepsia. He is


tested and found to be positive for
helicobacter pylori infection. One evening he has an episode of
haematemesis and collapses. What is the most
likely vessel to be responsible?
A. Portal vein
B. Short gastric arteries
C. Superior mesenteric artery
D. Gastroduodenal artery
E. None of the above
Theme from January 2012 exam

A 22 year old man is stabbed in the chest at the level of the junction
between the sternum and manubrium.
Which structure is at greatest risk?
A. Left atrium
B. Oesophagus
C. Thyroid gland
D. Inferior vena cava
E. Aortic arch
Theme from 2009 Exam

Which of the following nerves passes through the greater and lesser
sciatic foramina?
A. Pudendal nerve
B. Sciatic nerve
C. Superior gluteal nerve
D. Inferior gluteal nerve
E. Posterior cutaneous nerve of the thigh
Structures passing through the lesser and greater sciatic foramina
(medial to lateral): PIN
Pudendal nerve
Internal pudendal artery
Nerve to obturator internus
Theme from 2010 Exam
Theme from January 2013 Exam

A 73 year old lady is admitted with brisk rectal bleeding. Despite


attempts at resuscitation the bleeding proceeds
to cause haemodynamic compromise. An upper GI endoscopy is normal.
A mesenteric angiogram is performed
and a contrast blush is seen in the region of the sigmoid colon. The
radiologist decides to embolise the vessel
supplying this area. At what spinal level does it leave the aorta?
A. L2
B. L1
C. L4
D. L3
E. T10
Theme from 2009 Exam

In which space is a lumbar puncture performed?


A. Subdural space
B. Epidural space
C. Subarachnoid space
D. Extradural space
E. Intraventricular space
Theme from January 2013 exam

A 21 year old man is stabbed in the antecubital fossa. A decision is made


to surgically explore the wound. At
operation the surgeon dissects down onto the brachial artery. A nerve is
identified medially, which nerve is it
likely to be?
A. Radial
B. Recurrent branch of median
C. Anterior interosseous
D. Ulnar
E. Median
Theme from September 2012 Exam
A 65 year old man with long standing atrial fibrillation develops an
embolus to the lower leg. The decision is made to
perform an embolectomy, utilising a trans popliteal approach. After
incising the deep fascia, which of the following
structures will the surgeons encounter first on exploring the central
region of the popliteal fossa?
A. Popliteal vein
B. Common peroneal nerve
C. Popliteal artery
D. Tibial nerve
E. None of the above
Theme from 2009 Exam
Theme from January 2013 Exam

A man sustains a laceration between the base of the little finger and
wrist. Several weeks after the injury there is
loss of thumb adduction power. Which nerve is most likely to have been
injured?
A. Superficial ulnar nerve
B. Deep ulnar nerve
C. Median nerve
D. Radial nerve
E. Recurrent branch of median nerve
Theme from 2009 Exam

What is the course of the median nerve relative to the brachial artery in
the upper arm?
A. Medial to anterior to lateral
B. Lateral to posterior to medial
C. Medial to posterior to lateral
D. Medial to anterior to medial
E. Lateral to anterior to medial
Relations of median nerve to the
brachial artery:
Lateral -> Anterior -> Medial
Theme from 2009 and 2012 Exams

A 43 year old man is due to undergo an excision of the sub mandibular


gland. Which of the following incisions
is the most appropriate for this procedure?
A. A transversely orientated incision 4cm below the mandible
B. A transversely orientated incision immediately inferior to the
mandible
C. A vertical incision 3 cm anterior to the angle of the mandible and
extending inferiorly
D. A transversely orientated incision 2cm above the mandible
E. A transversely orientated incision 12cm below the mandible
Theme from 2009 Exam

A 5 year old boy presents with recurrent headaches. As part of his


assessment he undergoes an MRI scan of his
brain. This demonstrates enlargement of the lateral and third ventricles.
Where is the most likely site of
obstruction?
A. Foramen of Luschka
B. Foramen of Magendie
C. Foramen of Munro
D. Aqueduct of Sylvius
E. None of the above
Theme based on September 2011 Exam
Theme based on April 2012 Exam
Theme from September 2012 Exam

Where does the spinal cord terminate in neonates?


A. L1
B. L2
C. L3
D. L4
E. L5
Theme from 2009 Exam
Theme from January 2013 Exam
A 73 year old lady is admitted with acute mesenteric ischaemia. A CT
angiogram is performed and a stenotic
lesion is noted at the origin of the superior mesenteric artery. At which
of the following levels does this branch
from the aorta?
A. L1
B. L2
C. L3
D. L4
E. L5
Theme from January 2012 Exam

Following an oesophagogastrectomy the surgeons will anastomose the


oesophageal remnant to the stomach,
which of the following is not part of the layers that comprise the
oesophageal wall?
A. Serosa
B. Adventitia
C. Muscularis propria
D. Submucosa
E. Mucosa
The oesophageal wall lacks the serosa
layer
Theme from April 2010 exam

Where is the 'safe triangle' for chest drain insertion located?


A. 4th intercostal space, mid axillary line
B. 5th intercostal space, mid axillary line
C. 4th intercostal space, mid scapular line
D. 5th intercostal space, mid scapular line
E. 4th intercostal space, mid clavicular line
'Safe Triangle' for chest drain
insertion:
5th intercostal space, mid axillary line
Theme from April 2012 exam
Your consultant decides to perform an open inguinal hernia repair
under local anaesthesia. Which of the
following dermatomal levels will require blockade?
A. T10
B. T12
C. T11
D. S1
E. S2
Theme from April 2012 Exam

Which of the following does not pass through the superior


orbital fissure?
A. Inferior opthalmic vein
B. Abducens nerve
C. Opthalmic artery
D. Trochlear nerve
E. Superior opthalmic vein
Mnemonic for the nerves
passing through the
supraorbital fissure:
Live Frankly To See
Absolutely No Insult
Lacrimal
Frontal
Trochlear
Superior Division of
Oculomotor
Abducens
Nasociliary
Inferior Division of
Oculomotor nerve
Theme from January 2012 Exam

A patient is found to have an ischaemic left colon. Which artery arising


from the aorta at around the level of L3
is most likely to account for this situation?
A. Superior mesenteric artery
B. Inferior mesenteric artery
C. Superior rectal artery
D. Ileocolic artery
E. Middle colic artery
Theme from January 2013 Exam

At which level does the aorta perforate the diaphragm?


A. T10
B. T9
C. T8
D. T11
E. T12
Memory aid:
T8 (8 letters) = vena cava
T10 (10 letters) = oesophagus
T12 (12 letters) = aortic
hiatus
Theme from April 2012 exam

A 24 year old lady is stabbed in the buttock. Following the injury the
wound is sutured in the emergency
department. Eight weeks later she attends the clinic, as she walks into
the clinic room she has a waddling gait
and difficulty with thigh abduction. On examination she has buttock
muscle wasting. Which nerve has been
injured?
A. Superior gluteal nerve
B. Obturator nerve
C. Sciatic nerve
D. Femoral nerve
E. Inferior gluteal nerve
Theme from April 2012 Exam

At which level is the hilum of the left kidney located?


A. L1
B. L2
C. T12
D. T11
E. L3
Remember L1 ('left one') is the level of the
hilum of the left kidney
Theme from April 2012 exam

A 73 year old lady presents with symptoms of faecal incontinence. On


examination she has weak anal sphincter
muscles. What are the main nerve root values of the nerves supplying
the external anal sphincter?
A. S2,3
B. L5, S1
C. S4,5
D. S5
E. S2,3,4
Theme from September 2011 Exam

A 72 year old man has a fall. He is found to have a fractured neck of


femur and goes on to have a left hip
hemiarthroplasty. Two months post operatively he is found to have an
odd gait. When standing on his left leg
his pelvis dips on the right side. There is no foot drop. What is the
cause?
A. Sciatic nerve damage
B. L5 radiculopathy
C. Inferior gluteal nerve damage
D. Previous poliomyelitis
E. Superior gluteal nerve damage
Theme from 2010 Exam

A 28 year old lady requires an episiotomy for a ventouse vaginal


delivery. Which of the nerves listed below will
usually be anaesthetised to allow the episiotomy?
A. Femoral
B. Ilioinguinal
C. Pudendal
D. Genitofemoral
E. Sacral plexus
Theme from April 2013 Exam

A 48 year old lady is undergoing an axillary node clearance for breast


cancer. Which of the structures listed
below are most likely to be encountered during the axillary dissection?
A. Cords of the brachial plexus
B. Thoracodorsal trunk
C. Internal mammary artery
D. Thoracoacromial artery
E. None of the above
Beware of damaging the thoracodorsal trunk if a latissimus dorsi flap
reconstruction is planned.
Theme from 2009 Exam

A 53 year old lady is recovering following a difficult mastectomy and


axillary nodal clearance for carcinoma of
the breast. She complains of shoulder pain and on examination has
obvious winging of the scapula. Loss of
innervation to which of the following is the most likely underlying
cause?
A. Latissimus dorsi
B. Serratus anterior
C. Pectoralis minor
D. Pectoralis major
E. Rhomboids
Theme from April 2012 Exam

A 56 year old lady is referred to the colorectal clinic with symptoms of


pruritus ani. On examination a
polypoidal mass is identified inferior to the dentate line. A biopsy
confirms squamous cell carcinoma. To which
of the following lymph node groups will the lesion potentially
metastasise?
A. Internal iliac
B. External iliac
C. Mesorectal
D. Inguinal
E. None of the above
Theme from September 2011 Exam
Theme from April 2012 Exam

A 20 year old man is hit with a hammer on the right side of the head. He
dies on arrival in the emergency
department. Which of these features is most likely to be found at post
mortem?
A. Hydrocephalus
B. Supra tentorial herniation
C. Laceration of the middle meningeal artery
D. Sub dural haematoma
E. Posterior fossa haematoma
Theme based on 2011 exam

Which of the following ligaments contains the artery supplying the head
of femur in children?
A. Transverse ligament
B. Ligamentum teres
C. Iliofemoral ligament
D. Ischiofemoral ligament
E. Pubofemoral ligament
Theme from 2010 Exam

A 63 year old man is undergoing a coronary artery bypass procedure.


During the median sternotomy which
structure would routinely require division?
A. Parietal pleura
B. Interclavicular ligament
C. Internal mammary artery
D. Brachiocephalic vein
E. Left vagus nerve
Theme from January 2011 Exam

A man undergoes a high anterior resection for carcinoma of the upper


rectum. Which of the following vessels
will require ligation?
A. Superior mesenteric artery
B. Inferior mesenteric artery
C. Coeliac axis
D. Perineal artery
E. Middle colic artery
Theme from April 2013

What are the boundaries of the 'safe triangle' for chest drain insertion?
A. Bounded by trapezius, latissimus dorsi, and laterally by the vertebral
border of the scapula
B. Bounded by latissimus dorsi, pectoralis major, line superior to the
nipple and apex at the axilla
C. Bounded by latissimus dorsi, serratus anterior, line superior to the
nipple and apex at the axilla
D. Bounded by trapezius, deltoid, rhomboid major and teres minor
E. Bounded by trapezius, deltoid and latissimus dorsi
Theme from April 2012 Exam

A 60 year old female attends the preoperative hernia clinic. She reports
some visual difficulty. On examination
she is noted to have a homonymous hemianopia. Where is the lesion
most likely to be?
A. Frontal lobe
B. Pituitary gland
C. Parietal lobe
D. Optic chiasm
E. Optic tract
Lesions before optic chiasm:
Monocular vision loss = Optic nerve
lesion
Bitemporal hemianopia = Optic chiasm
lesion
Lesions after the optic chiasm:
Homonymous hemianopia = Optic tract
lesion
Upper quadranopia = Temporal lobe
lesion
Lower quadranopia = Parietal lobe
lesion
Theme from April 2012 exam

A 23 year old climber falls and fractures his humerus. The surgeons
decide upon a posterior approach to the
middle third of the bone. Which of the following nerves is at greatest
risk in this approach?
A. Ulnar
B. Antebrachial
C. Musculocutaneous
D. Radial
E. Intercostobrachial
Theme from April 2012 Exam

Following a carotid endarterectomy a man notices that he has a


weakness of his tongue. Damage to which of the
following nerves is the most likely explanation for this process?
A. Hypoglossal
B. Accessory
C. Ansa cervicalis
D. Vagus
E. Cervical plexus
Theme from April 2013

At which of the following levels does the inferior vena cava exit the
abdominal cavity?
A. T6
B. T7
C. T10
D. T8
E. T12
Theme from April 2012 Exam

Which of the following structures lies deepest in the popliteal fossa?


A. Popliteal artery
B. Popliteal vein
C. Tibial nerve
D. Common peroneal nerve
E. Popliteal lymph nodes
Theme from January 2012 exam

What is the nerve root value of the external urethral sphincter?


A. S4
B. S1, S2, S3
C. S2, S3, S4
D. L3, L4, L5
E. L5, S1, S2
Theme from April 2012 Exam

A patient presents to the clinic following a surgical procedure. She


complains that she is unable to shrug her
shoulder. What is the most likely underlying nerve injury?
A. Accessory nerve
B. Cervical plexus
C. Ansa cervicalis
D. Long thoracic nerve
E. Axillary nerve
Theme from April 2013 Exam

A 32 year old man is undergoing a splenectomy. Division of which of the


following will be necessary during
the procedure?
A. Left crus of diaphragm
B. Short gastric vessels
C. Gerotas fascia
D. Splenic flexure of colon
E. Marginal artery
Theme from 2011 Exam

A patient has a chest drain insertion. There is fresh blood at the chest
drain insertion area. Which vessel has
been damaged?
A. Pericardiophrenic artery
B. Intercostal vein
C. Right ventricle
D. Vagus artery
E. Intercostal artery
Theme from 2009 Exam

Two teenagers are playing with an airgun when one accidentally shoots
his friend in the abdomen. He is
brought to the emergency department. On examination there is a bullet
entry point immediately to the right of
the rectus sheath at the level of the 1st lumbar vertebra. Which of the
following structures is most likely to be
injured by the bullet?
A. Head of pancreas
B. Right ureter
C. Right adrenal gland
D. Fundus of the gallbladder
E. Gastric antrum
Theme from September 2011 Exam

Which of the following nerves is not contained within the posterior


triangle of the neck?
A. Accessory nerve
B. Phrenic nerve
C. Greater auricular nerve
D. Ansa cervicalis
E. Lesser occiptal nerve
Theme from September 2012 Exam

A 42 year old lady is reviewed in the outpatient clinic following a routine


surgical procedure. She complains of
diminished sensation at the lateral aspect of her foot. Which of the
following nerves is likely to be affected?
A. Sural
B. Superficial peroneal
C. Deep peroneal
D. Medial plantar
E. Lateral plantar
Theme from April 2012 Exam

A sprinter attends A&E with severe leg pain. He had forgotten to warm
up and ran a 100m sprint race. Towards
the end of the race he experienced pain in the posterior aspect of his
thigh. The pain worsens, localising to the
lateral aspect of the knee. The sprinter is unable to flex the knee. What
structure has been injured?
A. Anterior cruciate ligament
B. Posterior cruciate ligament
C. Semimembranosus tendon
D. Semiteninosus tendon
E. Biceps femoris tendon
Theme from 2009 Exam

Which nerve supplies the interossei of the fourth finger?


A. Radial
B. Median
C. Superficial ulnar
D. Deep ulnar
E. Posterior interosseous
Mnemonic:
PAD and DAB
Palmer interossei
ADduct
Dorsal interossei ABuct
Theme from April 2013 Exam

A 45 year man presents with hand weakness. He is given a piece of


paper to hold between his thumb and index
finger. When the paper is pulled, the patient has difficulty maintaining a
grip. Grip pressure is maintained by
flexing the thumb at the interphalangeal joint. What is the most likely
nerve lesion?
A. Posterior interosseous nerve
B. Deep branch of ulnar nerve
C. Anterior interosseous nerve
D. Superficial branch of the ulnar nerve
E. Radial nerve
Theme from January 2012 exam

Which of the following cranial foramina pairings are incorrect?


A. The foramen lacerum and internal carotid artery.
B. Foramen ovale and mandibular nerve.
C. Optic canal and ophthalmic artery.
D. Optic canal and ophthalmic nerve.
E. Foramen rotundum and maxillary nerve.
Question derived from 2010 and 2011 exams

A man has an incision sited than runs 8cm from the deltopectoral
groove to the midline. Which of the following
is not at risk of injury?
A. Cephalic vein
B. Shoulder joint capsule
C. Axillary artery
D. Pectoralis major
E. Trunk of the brachial plexus
Theme from April 2012 Exam
A surgeon is due to perform a laparotomy for perforated duodenal
ulcer. An upper midline incision is to be
performed. Which of the following structures is the incision most likely
to divide?
A. Rectus abdominis muscle
B. External oblique muscle
C. Linea alba
D. Internal oblique muscle
E. None of the above
Theme from September 2011 Exam

A 23 year old man is injured during a game of rugby. He suffers a


fracture of the distal third of his clavicle, it is
a compound fracture and there is evidence of arterial haemorrhage.
Which of the following vessels is most
likely to be encountered first during subsequent surgical exploration?
A. Posterior circumflex humeral artery
B. Axillary artery
C. Thoracoacromial artery
D. Sub scapular artery
E. Lateral thoracic artery
Similar theme in September 2011 Exam

What is the most useful test to clinically distinguish between an upper


and lower motor neurone lesion of the
facial nerve?
A. Blow cheeks out
B. Loss of chin reflex
C. Close eye
D. Raise eyebrow
E. Open mouth against resistance
Upper motor neurone lesions of the facial nerve- Paralysis of the lower
half of face.
Lower motor neurone lesion- Paralysis of the entire ipsilateral face.
Theme from April 2012 Exam

A 23 year old man is involved in a fight, during the dispute he sustains a


laceration to the posterior aspect of his
right arm, approximately 2cm proximal to the olecranon process. On
assessment in the emergency department
he is unable to extend his elbow joint. Which of the following tendons is
most likely to have been cut?
A. Triceps
B. Pronator teres
C. Brachioradialis
D. Brachialis
E. Biceps
Theme from 2009 Exam

During the course of a radical gastrectomy the surgeons detach the


omentum and ligate the right gastro-epiploic
artery. From which vessel does it originate?
A. Superior mesenteric artery
B. Inferior mesenteric artery
C. Coeliac axis
D. Common hepatic artery
E. Gastroduodenal artery
Theme from January 2013 Exam

A 43 year old man is reviewed in the clinic following a cardiac


operation. A chest x-ray is performed and a
circular radio-opaque structure is noted medial to the 4th interspace on
the left. Which of the following
procedures is the patient most likely to have undergone?
A. Aortic valve replacement with metallic valve
B. Tricuspid valve replacement with metallic valve
C. Tricuspid valve replacement with porcine valve
D. Pulmonary valve replacement with porcine valve
E. Mitral valve replacement with metallic valve
Theme from April 2012 Exam

A 63 year old lady is diagnosed as having an endometrial carcinoma


arising from the uterine body. To which
nodal region will the tumour initially metastasise?
A. Para aortic nodes
B. Iliac lymph nodes
C. Inguinal nodes
D. Pre sacral nodes
E. Mesorectal lymph nodes
Theme from 2011 exam

A 23 year old lady is undergoing a trendelenberg procedure for varicose


veins. During the dissection
of the saphenofemoral junction, which of the structures listed below is
most liable to injury?
A. Superficial circumflex iliac artery
B. Superficial circumflex iliac vein
C. Femoral artery
D. Femoral nerve
E. Deep external pudendal artery

Theme from September 2011 exam

Which of the following nerves is responsible for the motor innervation


of the sternocleidomastoid
muscle?
A. Ansa cervicalis
B. Accessory nerve
C. Hypoglossal nerve
D. Facial nerve
E. Vagus nerve
Theme from January 2013 Exam

A 42 year old lady has had an axillary node clearance for breast
malignancy. Post operatively she
reports weakness of the shoulder. She is unable to push herself forwards
from a wall with the right
arm and the scapula is pushed out medially from the chest wall. What is
the most likely nerve
injury?
A. C5, C6
B. C8, T1
C. Axillary nerve
D. Long thoracic nerve
E. Spinal accessory nerve

Theme from January 2012 and 2009 Exam

A 36 year old male is admitted for elective surgery for a lymph node
biopsy in the supraclavicular
region. Post operatively the patient has difficulty shrugging his left
shoulder. What nerve has been
damaged?
A. Phrenic nerve
B. Axillary nerve
C. C5, C6 lesion
D. C8, T1 lesion
E. Accessory nerve

Theme from September 2011 Exam

A 17 year old male presents to the clinic. He complains of difficulty


using his left hand. It has been a
persistent problem since he sustained a distal humerus fracture as a
child. On examination there is
diminished sensation overlying the hypothenar eminence and medial
one and half fingers. What is
the most likely nerve lesion?
A. Anterior interosseous nerve
B. Posterior interosseous nerve
C. Ulnar nerve
D. Median nerve
E. Radial nerve

Theme from April 2012 Exam

A 56 year old man is undergoing a pancreatectomy for carcinoma.


During resection of the gland
which of the following structures will the surgeon not encounter
posterior to the pancreas itself?
A. Left crus of the diaphragm
B. Superior mesenteric vein
C. Common bile duct
D. Portal vein
E. Gastroduodenal artery

Theme from 2010 Exam

Which of the following bones is related to the cuboid at its distal


articular surface?
A. All metatarsals
B. 5th metatarsal
C. Calcaneum
D. Medial cuneiform
E. 3rd metatarsal

Theme from April 2012 Exam

A 34 year old lady presents with symptoms of faecal incontinence. Ten


years previously she gave
birth to a child by normal vaginal delivery. Injury to which of the
following nerves is most likely to
account for this process?
A. Genitofemoral
B. Ilioinguinal
C. Pudendal
D. Hypogastric autonomic nerve
E. Obturator

Theme from April 2012 Exam

A 56 year old man undergoes an abdomino-perineal excision of the


rectum. He is assessed in the
outpatient clinic post operatively. His wounds are well healed. However,
he complains of impotence.
Which of the following best explains this problem?
A. Sciatic nerve injury
B. Damage to the internal iliac artery
C. Damage to the hypogastric nerve plexus
D. Damage to the vas
E. Damage to the genitofemoral nerve

Theme from 2012 Exam

A woman develops winging of the scapula following a Patey


mastectomy. What is the most likely
cause?
A. Division of pectoralis minor to access level 3 axillary nodes
B. Damage to the brachial plexus during axillary dissection
C. Damage to the long thoracic nerve during axillary dissection
D. Division of the thoracodorsal trunk during axillary dissection
E. Damage to the thoracodorsal trunk during axillary dissection

Theme from January 2012 exam


Which nerve directly innervates the sinoatrial node?
A. Superior cardiac nerve
B. Right vagus nerve
C. Left vagus nerve
D. Inferior cardiac nerve
E. None of the above

Theme from September 2011 Exam


Theme from September 2012 Exam

A 30 year old man presents with back pain and the surgeon tests the
ankle reflex. Which of the
following nerve roots are tested in this manoeuvre?
A. S3 and S4
B. L4 and L5
C. L3 and L4
D. S1 and S2
E. S4 only

Theme from April 2012 Exam

An 18 year old male presents to casualty with a depressed skull fracture.


This is managed surgically.
Over the next few days he complains of double vision on walking down
stairs and reading. On
examination the left eye cannot look downwards and medially. Which of
the nerves listed below is
most likely to be responsible?
A. Facial
B. Oculomotor
C. Abducens
D. Trochlear
E. Trigeminal nerve

Theme from September 2012 Exam

A 16 year old boy is hit by a car and sustains a blow to the right side of
his head. He is initially
conscious but on arrival in the emergency department is comatose. On
examination his right pupil is
fixed and dilated. The neurosurgeons plan immediate surgery. What
type of initial approach should
be made?
A. Left parieto-temporal craniotomy
B. Right parieto-temporal craniotomy
C. Posterior fossa craniotomy
D. Left parieto-temporal burr holes
E. None of the above

Theme from April 2012 Exam

A 56 year old man is having a long venous line inserted via the femoral
vein into the right atrium for
CVP measurements. The catheter is advanced through the IVC. At
which of the following levels does
this vessel enter the thorax?
A. L2
B. T10
C. L1
D. T8
E. T6

Theme from 2010 Exam

A 23 year old man falls and injures his hand. There are concerns that he
may have a scaphoid
fracture as there is tenderness in his anatomical snuffbox on clinical
examination. Which of the
following forms the posterior border of this structure?
A. Basilic vein
B. Radial artery
C. Extensor pollicis brevis
D. Abductor pollicis longus
E. Extensor pollicis longus

Theme from 2009 Exam


Theme from September 2012 Exam

The integrity of which muscle is assessed by the Trendelenberg test?


A. Sartorius
B. Quadratus femoris
C. Semimembranosus
D. Gluteus medius
E. Piriformis

Theme from 2011 Exam

Which of the following regions of the male urethra is entirely


surrounded by Bucks fascia?
A. Preprostatic part
B. Prostatic part
C. Membranous part
D. Spongiose part
E. None of the above

Theme from 2010 Exam

A 73 year old man has a large abdominal aortic aneurysm. During a


laparotomy for planned surgical
repair the surgeons find the aneurysm is far more proximally located
and lies near the origin of the
SMA. During the dissection a vessel lying transversely across the aorta
is injured. What is this vessel
most likely to be?
A. Left renal vein
B. Right renal vein
C. Inferior mesenteric artery
D. Ileocolic artery
E. Middle colic artery

Theme from April 2012 Exam

A 56 year old machinist has his arm entrapped in a steel grinder and is
brought to the emergency
department. On examination, he is unable to extend his
metacarpophalangeal joints and abduct his
shoulder. He has weakness of his elbow and wrist. What has been
injured?
A. Ulnar nerve
B. Axillary nerve
C. Medial cord of brachial plexus
D. Lateral cord of brachial plexus
E. Posterior cord of brachial plexus

The posterior cord gives rise to:


Radial nerve ((innervates the triceps, brachioradialis, wrist
extensors, and finger extensors)
Axillary nerve (innervates deltoid and teres minor)
Upper subscapular nerve (innervates subscapularis)
Lower subscapular nerve (innervates teres major and
subscapularis)
Thoracodorsal nerve (innervates latissimus dorsi)
Theme from September 2012 exam

A 60 year old female is undergoing a Whipples procedure for


adenocarcinoma of the pancreas. As
the surgeons begin to mobilise the pancreatic head they identify a large
vessel passing inferiorly
over the anterior aspect of the pancreatic head. What is it likely to be?
A. Superior mesenteric artery
B. Coeliac axis
C. Inferior mesenteric artery
D. Aorta
E. Left gastric artery

Theme from January 2012 Exam

Which of the following structures does not lie posterior to the right
kidney?
A. Psoas major
B. Transversus abdominis
C. Quadratus lumborum
D. Medial artcuate ligament
E. 10th rib

Theme from April 2012 Exam

Which of the following muscles is not within the posterior compartment


of the lower leg?
A. Peroneus brevis
B. Flexor digitalis longus
C. Soleus
D. Popliteus
E. Flexor hallucis longus

Theme from 2007 Exam

A 20 year old man undergoes an open appendicectomy performed via a


lanz incision. This surgeon
places the incision on a level of the anterior superior iliac spine in an
attempt to improve cosmesis.
During the procedure the appendix is found to be retrocaecal and the
incision is extended laterally.
Which of the following nerves is at greatest risk of injury?
A. Genitofemoral
B. Ilioinguinal
C. Obturator
D. Lateral femoral cutaneous
E. Femoral

Theme from April 2012 Exam

A 32 year old man is stabbed in the neck and the inferior trunk of his
brachial plexus is injured.
Which of the modalities listed below is least likely to be affected?
A. Initiating abduction of the shoulder
B. Abduction of the fingers
C. Flexion of the little finger
D. Sensation on the palmar aspect of the little finger
E. Gripping a screwdriver

Inferior trunk of brachial plexus.


C8 and T1 roots
Contributes to ulnar nerve and part of median nerve
Theme from September 2012 Exam

A 23 year old man presents with delayed diagnosis of appendicitis. The


appendix is retrocaecal and
has perforated causing a psoas abscess. Into which structure does the
psoas major muscle insert?
A. Greater trochanter of the femur
B. Linea aspera of the femur
C. Lesser trochanter of the femur
D. Iliac crest
E. None of the above

Theme based on 2011 exam


A 63 year old man is due to undergo a splenectomy. Which splenic
structure lies most posteriorly?
A. Gastrosplenic ligament
B. Splenic vein
C. Splenic artery
D. Splenic notch
E. Lienorenal ligament

Theme from 2011 Exam

A 62 year old man presents with arm weakness. On examination he has


a weakness of elbow
extension and loss of sensation on the dorsal aspect of the first digit.
What is the site of the most
likely underlying defect?
A. Axillary nerve
B. Median nerve
C. Ulnar nerve
D. Radial nerve
E. Musculocutaneous nerve

Theme from April 2012 Exam

Theme: Nerve injury


A. Median nerve
B. Ulnar nerve
C. Radial nerve
D. Anterior interosseous nerve
E. Posterior interosseous nerve
F. Axillary nerve
G. Musculocutaneous nerve
Please select the nerve at risk of injury in each scenario. Each option
may be used once, more than
once or not at all.
44. A 43 year old typist presents with pain at the dorsal aspect of the
upper part of her forearm. She also
complains of weakness when extending her fingers. On examination
triceps and supinator are both
functioning normally. There is weakness of most of the extensor
muscles. However, there is no sensory
deficit.
You answered Radial nerve
The correct answer is Posterior interosseous nerve
The radial nerve may become entrapped in the "arcade of Frohse"
which is a superficial part of the
supinator muscle which overlies the posterior interosseous nerve. This
nerve is entirely muscular and
articular in its distribution. It passes postero-inferiorly and gives
branches to extensor carpi radialis
brevis and supinator. It enters supinator and curves around the lateral
and posterior surfaces of the
radius. On emerging from the supinator the posterior interosseous
nerve lies between the superficial
extensor muscles and the lowermost fibres of supinator. It then gives
branches to the extensors.
45. A 28 year teacher reports difficulty with writing. There is no sensory
loss. She is known to have an
aberrant Gantzer muscle.
You answered Posterior interosseous nerve
The correct answer is Anterior interosseous nerve
Anterior interosseous lesions occur due to fracture, or rarely due to
compression. The Gantzer muscle is
an aberrant accessory of the flexor pollicis longus and is a risk factor for
anterior interosseous nerve
compression. Remember loss of pincer grip and normal sensation
indicates an interosseous nerve lesion.
46. A 35 year tennis player attends reporting tingling down his arm. He
says that his 'funny bone' was hit
very hard by a tennis ball. There is weakness of abduction and
adduction of his extended fingers.
You answered Musculocutaneous nerve
The correct answer is Ulnar nerve
Theme from September 2012 exam
The ulnar nerve arises from the medial cord of the brachial plexus (C8,
T1 and contribution from C7).
The nerve descends between the axillary artery and vein, posterior to
the cutaneous nerve of the forearm
and then lies anterior to triceps on the medial side of the brachial
artery. In the distal half of the arm it
passes through the medial intermuscular septum, and continues
between this structure and the medial
head of triceps to enter the forearm between the medial epicondyle of
the humerus and the olecranon. It
may be injured at this site in this scenario

In relation to the middle cranial fossa, which of the following statements


relating to the foramina are
incorrect?
A. The foramen rotundum transmits the maxillary nerve
B. The foramen lacerum transmits the internal carotid artery
C. The foramen spinosum lies posterolateral to the foramen ovale
D. The foramen ovale transmits the middle meningeal artery
E. The foramen rotundum lies anteromedial to the foramen ovale

Theme addressed in 2010 and 2011 exam

During an operation for varicose veins the surgeons are mobilising the
long saphenous vein. Near its
point of entry to the femoral vein an artery is injured and bleeding is
encountered. From where is
the bleeding most likely to originate?
A. Femoral artery
B. Profunda femoris artery
C. Superficial circumflex iliac artery
D. Superficial epigastric artery
E. Deep external pudendal artery

Theme from 2011 Exam


What is embryological origin of the pulmonary artery?
A. First pharyngeal arch
B. Second pharyngeal arch
C. Fourth pharyngeal arch
D. Fifth pharyngeal arch
E. Sixth pharyngeal arch

Theme from September 2011 Exam


Theme from September 2012 Exam

A 53 year old lady presents with pain and discomfort in her hand. She
works as a typist and notices
that the pain is worst when she is working. She also suffers symptoms at
night. Her little finger is
less affected by the pain. Which of the nerves listed below is most likely
to be affected?
A. Radial
B. Median
C. Ulnar
D. Anterior interosseous nerve
E. Posterior interosseous nerve

Motor supply: LOAF


L ateral 2 lumbricals
O pponens pollicis
A bductor pollicisbrevis
F lexor pollicis brevis
Theme from April 2012 Exam

Which of the following muscles lies medial to the long thoracic nerve?
A. Serratus anterior
B. Latissimus dorsi
C. Pectoralis major
D. Pectoralis minor
E. None of the above

Theme from 2009 Exam

A 25 year old man is being catheterised, prior to a surgical procedure.


As the catheter enters the
prostatic urethra which of the following changes will occur?
A. Resistance will increase significantly
B. Resistance will increase slightly
C. It will lie horizontally
D. Resistance will decrease
E. It will deviate laterally

Theme from September 2011 Exam

Which of the following nerves is the primary source of innervation to


the anterior scrotal skin?
A. Genital branch of the genitofemoral nerve
B. Pudendal nerve
C. Ilioinguinal nerve
D. Femoral branch of the genitofemoral nerve
E. Obturator nerve

Theme from April 2012 Exam

A 38 year old man presents to the clinic with shoulder weakness. On


examination he has an inability to initiate shoulder
abduction. Which of the nerves listed below is least likely to be
functioning normally?
A. Suprascapular nerve
B. Medial pectoral nerve
C. Axillary nerve
D. Median nerve
E. Radial nerve
Theme from April 2012 Exam

A 72 year old lady with osteoporosis falls and sustains an intracapsular


femoral neck fracture. The fracture is completely
displaced. Which of the following vessels is the main contributor to the
arterial supply of the femoral head?
A. Deep external pudendal artery
B. Superficial femoral artery
C. External iliac artery
D. Circumflex femoral arteries
E. Superficial external pudendal artery

Theme from 2010 Exam

A 21 year old man is hit with a hammer and sustains a depressed skull
fracture at the vertex. Which of the following sinuses is
at risk in this injury?
A. Superior sagittal sinus
B. Inferior petrosal sinus
C. Transverse sinus
D. Inferior sagittal sinus
E. Straight sinus

Theme in September 2011 Exam

A 19 year old man is playing rugby when he suddenly notices a severe


pain at the posterolateral aspect of his right thigh. Which
of the following muscle groups is most likely to have been injured?
A. Semimembranosus
B. Semitendinosus
C. Long head of biceps femoris
D. Gastrocnemius
E. Soleus

Theme from April 2012 Exam


--------------------------------------------------------------------------------------------
--------------------------------------------------------------------------------------------
--------------------------------------------------------------------------------------------
--------------------------------------------------------------------------------------------
--------------------------------------------------------------------------------------------
--------------------------------
A 23 year old is stabbed in the groin and develops hypovolaemic shock.
What is the most likely finding
on analysis of his urine?
A. Decreased specific gravity
B. Increased specific gravity
C. Increased urinary glucose
D. Increased urinary protein
E. Increased red blood cells in the urine
Theme from April 2013 Exam

A 45 year old male is diagnosed with carcinoma of the head of the


pancreas. He reports that his stool
sticks to the commode and will not flush away. Loss of which of the
following enzymes is most likely to
be responsible for this problem?
A. Lipase
B. Amylase
C. Trypsin
D. Elastase
E. None of the above
Theme from April 2012 Exam

A 34 year old man receives morphine following an appendicectomy. He


develops constipation as a
result. Which of the following best accounts for this process?
A. Stimulation of DOPA receptors
B. Inhibition of DOPA receptors
C. Stimulation of μ receptors
D. Stimulation of serotonin release
E. Inhibition of serotonin release
4 Types of opioid receptor:
δ (located in CNS)- Accounts for analgesic and antidepressant
effects
k (mainly CNS)- analgesic and dissociative effects
μ (central and peripheral) - causes analgesia, miosis, decreased gut
motility
Nociceptin receptor (CNS)- Affect of appetite and tolerance to μ
agonists.
Theme from April 2013 Exam

A 56 year old male presents to the acute surgical take with severe
abdominal pain. He is normally fit and
well. He has no malignancy. The biochemistry laboratory contacts the
ward urgently, his corrected
calcium result is 3.6 mmol/l. What is the medication of choice to treat
this abnormality?
A. IV Pamidronate
B. Oral Alendronate
C. Dexamethasone
D. Calcitonin
E. IV Zoledronate
Theme from January 2012 exam

An over enthusiastic medical student decides to ask you questions about


ECGs. Rather than admitting
your dwindling knowledge on this topic, you bravely attempt to answer
her questions! One question is
what segment of the ECG represents ventricular repolarization?
A. QRS complex
B. Q-T interval
C. P wave
D. T wave
E. S-T segment
Theme from January 2012 exam
A 28 year old man is shot in the abdomen and haemorrhages. Which of
the following substances will
produce vasoconstriction in response to this process?
A. Renin
B. Angiotensin I
C. Angiotensin II
D. Aldosterone
E. None of the above
Similar theme to September 2011 Exam

A 43 year old lady is recovering on the intensive care unit following a


Whipples procedure. She has a
central venous line in situ. Which of the following will lead to the "y"
descent on the waveform trace?
A. Ventricular contraction
B. Emptying of the right atrium
C. Emptying of the right ventricle
D. Opening of the pulmonary valve
E. Cardiac tamponade
JVP
3 Upward deflections and 2 downward
deflections
Upward deflections
a wave = atrial contraction
c wave = ventricular contraction
v wave = atrial venous filling
Downward deflections
x wave = atrium relaxes and tricuspid
valve moves down
y wave = ventricular filling
Theme from January 2012

A 25 year old man is undergoing respiratory spirometry. He takes a


maximal inspiration and maximally
exhales. Which of the following measurements will best illustrate this
process?
A. Functional residual capacity
B. Vital capacity
C. Inspiratory capacity
D. Maximum voluntary ventilation
E. Tidal volume
Theme from April 2012 Exam

What is the typical stroke volume in a resting 70 Kg man?


A. 10ml
B. 150ml
C. 125ml
D. 45ml
E. 70ml
Theme from April 2012 Exam

A 25 year old man undergoes an appendicetomy for appendicitis. The


appendix is submitted for
histopathological evaluation. Which of the following is most likely to be
identified microscopically?
A. Macrophages
B. Neutrophils
C. Fibroblasts
D. Lymphocytes
E. Stem cells
Theme from April 2013 Exam

A 73 year old man has an arterial line in situ. On studying the trace the
incisura can be seen. What is the
physiological event which accounts for this process?
A. Atrial repolarisation
B. Mitral valve closure
C. Ventricular repolarisation
D. Elastic recoil of the aorta
E. Tricuspid valve closure
Theme from 2010 Exam
It is the temporary rise in aortic pressure occurring as a result of elastic
recoil
A 22 year old man suffers a blunt head injury. He is drowsy and has a
GCS of 7 on admission. Which of
the following is the major determinant of cerebral blood flow in this
situation?
A. Systemic blood pressure
B. Mean arterial pressure
C. Intra cranial pressure
D. Hypoxaemia
E. Acidosis
Theme from 2009 Exam
Hypoxaemia and acidosis may both affect cerebral blood flow. However,
in the traumatic situation
increases in intracranial pressure are far more likely to occur especially
when GCS is low. This will
adversely affect cerebral blood flow.

A 43 year old man has recurrent episodes of dyspepsia and treatment is


commenced with oral antacids.
Which of the hormones listed below is released in response to increased
serum gastrin levels and
decreases intra gastric pH?
A. Cholecystokinin
B. Histamine
C. Somatostatin
D. Insulin
E. Vasoactive intestinal peptide
Theme from January 2013 Exam

Which of the following cell types is least likely to be found in a wound 1


week following injury?
A. Macrophages
B. Fibroblasts
C. Myofibroblasts
D. Endothelial cells
E. Neutrophils
Theme from April 2012 Exam

Which of the following best accounts for the action of PTH in increasing
serum calcium levels?
A. Activation of vitamin D to increase absorption of calcium from the
small intestine.
B. Direct stimulation of osteoclasts to absorb bone with release of
calcium.
C. Stimulation of phosphate absorption at the distal convoluted tubule
of
the kidney.
D. Decreased porosity of the vessels at Bowmans capsule to calcium.
E. Vasospasm of the afferent renal arteriole thereby reducing GFR and
calcium urinary loss.

Theme from April 2012 Exam
PTH increases the activity of 1-α-hydroxylase enzyme, which converts
25-
hydroxycholecalciferol to 1,25-dihydroxycholecalciferol, the active form
of vitamin D.
Osteoclasts do not have a PTH receptor and effects are mediated via
osteoblasts.

What is the most likely cause for this patients deterioration?


A. Acute respiratory alkalosis secondary to hyperventilation
B. Over administration of oxygen in a COPD patient
C. Metabolic acidosis secondary to severe pancreatitis
D. Metabolic alkalosis secondary to hypokalaemia
E. Acute respiratory acidosis secondary to pneumonia

Theme from April 2012 exam
A 43 year old man has a nasogastric tube inserted. The nurse takes a
small aspirate of the fluid
from the stomach and tests the pH of the aspirate. What is the normal
intragastric pH?
A. 0.5
B. 2
C. 4
D. 5
E. 6

Theme from January 2012 Exam

Theme: Critical care


A. Hypovolaemia
B. Normal
C. Cardiogenic shock
D. Septic shock
For each of the scenarios outlined in the tables below, please select the
most likely diagnosis from the
list. Each option may be used once, more than once or not at all.
51. A 45 year old man is admitted to the intensive care unit following a
laparotomy. He has a central
line, pulmonary artery catheter and arterial lines inserted. The
following results are obtained:
Pulmonary artery occlusion pressure Cardiac output Systemic vascular
resistance
Low Low High
Hypovolaemia
Theme from April 2012 Exam
Cardiac output is lowered in hypovolaemia due to decreased preload.
52. A 75 year old man is admitted to the intensive care unit following a
laparotomy. He has a central
line, pulmonary artery catheter and arterial lines inserted. The
following results are obtained:
Pulmonary artery occlusion pressure Cardiac output Systemic vascular
resistance
High Low High
Cardiogenic shock
In cardiogenic shock pulmonary pressures are often high. This is the
basis for the use of
venodilators in the treatment of pulmonary oedema.
53. A 22 year old lady is admitted to the intensive care unit following a
laparotomy. She has a central
line, pulmonary artery catheter and arterial lines inserted. The
following results are obtained:
Pulmonary artery occlusion pressure Cardiac output Systemic vascular
resistance
Low High Low
Septic shock
Decreased SVR is a major feature of sepsis. A hyperdynamic circulation
is often present. This is
the reason for the use of vasoconstrictors.

A 22 year old lady receives intravenous morphine for acute abdominal


pain. Which of the following best
accounts for its analgesic properties?
A. Binding to δ opioid receptors in the brainstem
B. Binding to δ opioid receptors at peripheral nerve sites
C. Binding to β opioid receptors within the CNS
D. Binding to α opioid receptors within the CNS
E. Binding to μ opioid receptors within the CNS
4 Types of opioid receptor:
δ (located in CNS)- Accounts for analgesic and antidepressant
effects
k (mainly CNS)- analgesic and dissociative effects
μ (central and peripheral) - causes analgesia, miosis, decreased gut
motility
Nociceptin receptor (CNS)- Affect of appetite and tolerance to μ
agonists.
Theme from April 2012 Exam
Which of the following areas is predominantly concerned with
thermoregulation?
A. Hypothalamus
B. Anterior pituitary
C. Cerebellum
D. Brain stem
E. Temporal lobe
Theme from 2012 Exam

A 19 year old man is attacked outside a club and beaten with a baseball
bat. He sustains a blow to the
right side of his head. He is brought to the emergency department and a
policy of observation is adopted.
His glasgow coma score deteriorates and he becomes comatose. Which
of the following haemodynamic
parameters is most likely to be present?
A. Hypertension and bradycardia
B. Hypotension and tachycardia
C. Hypotension and bradycardia
D. Hypertension and tachycardia
E. Normotension and bradycardia
Theme from April 2013 Exam

Which substance can be used to achieve the most accurate measurement


of the glomerular filtration
rate?
A. Glucose
B. Protein
C. Inulin
D. Creatinine
E. Para-amino hippuric acid
Theme from January 2013 exam
Creatinine declines with age due to decline in renal function and muscle
mass. Glucose, protein (amino
acids) and PAH are reabsorbed by the kidney.
A 17 year old lady with long standing anorexia nervosa is due to
undergo excision of a lipoma. Which of
the following nutritional deficiencies is most likely to be implicated in
poor collagen formation as the
wound heals?
A. Deficiency of copper
B. Deficiency of iron
C. Deficiency of ascorbic acid
D. Deficiency of phosphate
E. None of the above
Theme from January 2011

A 45 year old man is undergoing a small bowel resection. The


anaesthetist decides to administer an
intravenous fluid which is electrolyte rich. Which of the following most
closely matches this
requirement?
A. Dextrose / Saline
B. Pentastarch
C. Gelofusine
D. Hartmans
E. 5% Dextrose with added potassium 20mmol/ L
Theme from April 2012 Exam

A 16 year old girl develops pyelonephritis and is admitted in a state of


septic shock. Which of
the following is not typically seen in this condition?
A. Increased cardiac output
B. Increased systemic vascular resistance
C. Oliguria may occur
D. Systemic cytokine release
E. Tachycardia

Cardiogenic Shock:
e.g. MI, valve abnormality
increased SVR (vasoconstriction in response to low BP)
increased HR (sympathetic response)
decreased cardiac output
decreased blood pressure
Hypovolaemic shock:
blood volume depletion
e.g. haemorrhage, vomiting, diarrhoea, dehydration, third-space losses
during major operations
increased SVR
increased HR
decreased cardiac output
decreased blood pressure
Septic shock:
occurs when the peripheral vascular dilatation causes a fall in SVR
similar response may occur in anaphylactic shock, neurogenic shock
reduced SVR
increased HR
normal/increased cardiac output
decreased blood pressure

Theme from January 2012 Exam
The SVR is reduced in sepsis and for this reason a vasoconstricting
inotrope such as
noradrenaline may be used if hypotension and oliguria remain a
concern despite administration
of adequate amounts of intravenous fluids.

Which receptor does noradrenaline mainly bind to?


A. α 1 receptors
B. α 2 receptors
C. β 1 receptors
D. β 2 receptors
E. G receptors

Theme from 2009 Exam
--------------------------------------------------------------------------------------------
--------------------------------------------------------------------------------------------
--------------------------------------------------------------------------------------------
--------------------------------------------------------------------------------------------
--------------------------------------------------------------------------------

A 12 year old child is admitted with a 12 hour history of colicky right


upper quadrant pain. On
examination the child is afebrile and is jaundiced. The abdomen is soft
and non tender at the time of
examination. What is the most likely cause?
A. Infectious hepatitis
B. Acute cholecystitis
C. Cholangitis
D. Hereditary spherocytosis
E. Gilberts syndrome
Theme from September 2012 Exam

A 40 year old man presents with obstructive jaundice. Twenty years


previously he underwent a right
hemicolectomy for a mucinous right sided colonic carcinoma. He was
subsequently diagnosed as having
Lynch syndrome. What is the most likely cause of his jaundice?
A. Hepatocellular carcinoma
B. Liver metastasis from colonic cancer
C. Pancreatic carcinoma
D. Duodenal carcinoma
E. Gastric carcinoma
Theme from April 2013

Theme: Breast disease


A. Tuberculosis
B. Actinomycosis
C. Duct ectasia
D. Fibroadenoma
E. Fat necrosis
F. Intraductal papilloma
G. Breast abscess
H. Breast cancer
What is the most likely diagnosis for each scenario given? Each
diagnosis may be used once, more than
once or not at all.
6. A 32 year old woman presents with a tender breast lump. She has a 2
month old child. Clinically
there is a tender, fluctuant mass of the breast.
Breast abscess
Theme from January 2013 Exam
Theme from September 2011 Exam
This lady is likely to be breast feeding and is at risk of mastitis. This
may lead to an abscess if not
treated. Staphylococcus aureus is usually the causative organism.
7. A 53 year old lady presents with a creamy nipple discharge. On
examination she has discharge
originating from multiple ducts and associated nipple inversion.
Duct ectasia
Duct ectasia is common during the period of breast involution that
occurs during the menopausal
period. As the ducts shorten they may contain insipiated material. The
discharge will often
discharge from several ducts.
8. A 52 year old lady presents with an episode of nipple discharge. It is
usually clear in nature. On
examination the discharge is seen to originate from a single duct and
although it appears clear,
when the discharge is tested with a labstix it is shown to contain blood.
Imaging and examination
shows no obvious mass lesion.
Intraductal papilloma
Intraductal papilloma usually cause single duct discharge. The fluid is
often clear, although it may
be blood stained. If the fluid is tested with a labstix (little point in
routine practice) then it will
usually contain small amounts of blood. A microdocechtomy may be
performed.
A 2 day old baby is noted to have voiding difficulties and on closer
inspection is noted to have
hypospadias. Which of the following abnormalities is most commonly
associated with the condition?
A. Cryptorchidism
B. Diaphragmatic hernia
C. Ventricular - septal defect
D. Bronchogenic cyst
E. Atrial septal defect
Theme from January 2012 Exam
Hypospadias most commonly occurs as an isolated disorder. Associated
urological abnormalities may be
seen in up to 40% of infants, of these cryptorchidism is the most
frequent (10%).

Theme: Liver lesions


A. Cystadenoma
B. Hydatid cyst
C. Amoebic abscess
D. Mesenchymal hamartoma
E. Liver cell adenoma
F. Cavernous haemangioma
Please select the most likely lesion for the scenario given. Each option
may be used once, more than
once or not at all.
10. A 38 year old lady presents with right upper quadrant pain and
nausea. She is otherwise well and
her only medical therapy is the oral contraceptive pill which she has
taken for many years with no
ill effects. Her liver function tests are normal. An ultrasound
examination demonstrates a
hyperechoic well defined lesion in the left lobe of the liver which
measures 14 cm in diameter.
You answered Liver cell adenoma
The correct answer is Cavernous haemangioma
Cavernous haemangioma often presents with vague symptoms and
signs. They may grow to
considerable size. Liver function tests are usually normal. The lesions
are typically well defined
and hyperechoic on ultrasound. A causative link between OCP use and
haemangiomata has yet to
be established, but is possible.
11. A 37 year old lady presents with right upper quadrant pain and
nausea. She is otherwise well and
her only medical therapy is the oral contraceptive pill which she has
taken for many years with no
ill effects. Her liver function tests and serum alpha feto protein are
normal. An ultrasound
examination demonstrates a 4cm non encapsulated lesion in the right
lobe of the liver which has a
mixed echoity and heterogeneous texture.
You answered Cystadenoma
The correct answer is Liver cell adenoma
Liver cell adenomas are linked to OCP use and 90% of patients with
liver cell adenomas have
used the OCP. Liver function tests are often normal. The lesions will
typically have a mixed
echoity and heterogeneous texture.
12. A 38 year old shepherd presents to the clinic with a 3 month history
of malaise and right upper
quadrant pain. On examination he is mildly jaundiced. His liver
function tests demonstrate a mild
elevation in bilirubin and transaminases, his full blood count shows an
elevated eosinophil level.
An abdominal x-ray is performed by the senior house officer and
demonstrates a calcified lesion
in the right upper quadrant of the abdomen.
Hydatid cyst
Similar theme in September 2011 Exam
Hyatid disease is more common in those who work with sheep or dogs.
Liver function tests may
be abnormal and an eosinophilia is often present. Plain radiographs
may reveal a calcified cyst
wall.
A 5 year old boy presents to the clinic with short stature suggestive of
achondroplasia. What is the
genetic basis of this condition?
A. X linked defect
B. Y linked defect
C. YY linked defect
D. Autosomal dominant defect
E. Autosomal recessive defect
Theme from April 2013 Exam

A 58 year old man has been suffering from mechanical back pain for
several years. One morning he
awakes from sleep and feels a sudden onset of pain in his back radiating
down his left leg. Which of the
following events is most likely to account for his symptoms?
A. Prolapse of inner annulus fibrosus
B. Prolapse of outer annulus fibrosus
C. Prolapse of nucleus pulposus
D. Rupture of the ligamentum flavum
E. None of the above
Theme from 2009 Exam
Theme from September 2012 Exam

A 34 year old man presents to the surgical clinic 8 months following a


laparotomy for a ruptured spleen.
He complains of a nodule in the centre of his laparotomy wound. This is
explored surgically and a stitch
granuloma is found and excised. From which of the following cell types
do granulomata arise?
A. Polymorpho nucleocytes
B. Plasma cells
C. Reed- Sternberg cells
D. Platelets
E. Macrophages
Granulomas are organised collections
of macrophages
Theme from 2011 Exam

A 72 year old man has just undergone an emergency repair for a


ruptured abdominal aortic
aneurysm. Pre operatively he was taking aspirin, clopidogrel and
warfarin. Intra operatively he
received 5000 units of unfractionated heparin prior to application of the
aortic cross clamp. His
blood results on admission to the critical care unit are as follows:
Full blood count
Hb 8 g/dl
Platelets 40 * 109/l
WBC 7.1 * 109/l
His fibrin degradation products are measured and found to be
markedly elevated. Which of the
following accounts for these results?
A. Anastomotic leak
B. Disseminated intravascular coagulation
C. Heparin induced thrombocytopenia
D. Adverse effect of warfarin
E. Adverse effects of antiplatelet agents

Theme from April 2012 Exam

A 23 year old man presents to the surgical clinic with an inguinal hernia.
On examination he has a small
direct hernia. However, you also notice that he has pigmented spots
around his mouth, on his palms and
soles. In his history he underwent a reduction of an intussusception aged
12 years. Which of the
following lesions is most likely to be identified if a colonoscopy were
performed?
A. Hamartomas
B. Tubulovillous adenoma
C. Colorectal cancer
D. Crohns disease
E. Hyperplastic polyps
Theme from April 2012 Exam
Theme from January 2013 Exam
He is most likely to have Peutz-Jeghers syndrome which is associated
with Hamartomas.

A 56 year old surgeon has been successfully operating for many years.
Over the past few weeks she has
begun to notice that her hands are becoming blistering and weepy. A
latex allergy is diagnosed. Which
of the following pathological processes accounts for this scenario?
A. Type 1 hypersensitivity reaction
B. Type 2 hypersensitivity reaction
C. Type 4 hypersensitivity reaction
D. Type 3 hypersensitivity reaction
E. None of the above
Hypersensitivity reactions:
ACID
type 1 --Anaphylactic
type 2 --Cytotoxic
type 3 --Immune complex
type 4 --Delayed hypersensitivity
Theme from 2012 Exam

A 43 year old man presents with haemoptysis and is diagnosed as having


tuberculosis. Which of the cell
types listed below will usually internalise the tubercule bacullis?
A. Fibroblast
B. Neutrophil
C. Erythrocyte
D. Macrophage
E. Eosinophil
Theme from January 2013 Exam

Which of these tumour markers is most helpful in identifying an


individual with hepatocellular
carcinoma?
A. Serum AFP
B. Serum CA19-9
C. CEA
D. Beta HCG
E. CA125
Theme from September 2011 Exam

Theme: Renal stones


A. Calcium oxalate
B. Uric acid
C. Cystine
D. Struvite
E. Calcium phosphate
Please select the most likely stone type for each of the following urinary
tract stone scenarios. Each
option may be used once, more than once or not at all.
34. A 73 year old lady is undergoing chemotherapy for treatment of
acute leukaemia. She develops
symptoms of renal colic. Her urine tests positive for blood. A KUB x-ray
shows no evidence of
stones.
Uric acid
Chemotherapy and cell death can increase uric acid levels. In this acute
setting the uric acid
stones are unlikely to be coated with calcium and will therefore be
radiolucent.
35. A 16 year old boy presents with renal colic. His parents both have a
similar history of the
condition. His urine tests positive for blood. A KUB style x-ray shows a
relatively radiodense
stone in the region of the mid ureter.
You answered Struvite
The correct answer is Cystine
Cystine stones are associated with an inherited metabolic disorder.
36. A 43 year old lady with episodes of recurrent urinary tract sepsis
presents with a staghorn
calculus of the left kidney. Her urinary pH is 7.3. A KUB x-ray shows a
faint outline of the
calculus.
You answered Calcium phosphate
The correct answer is Struvite
Theme from April 2012 Exam
Chronic infection with urease producing enzymes can produce an
alkaline urine with formation of
struvite stone.

A pathologist is examining a histological section and identifies Hassall's


corpuscles. With what are they
most commonly associated?
A. Follicular carcinoma of the thyroid
B. Medulla of the thymus
C. Medulla of the spleen
D. Medulla of the kidney
E. Fundus of the stomach
Theme from 2010 Exam
Theme from January 2013 Exam
Theme from April 2013 Exam

A 32 year old man is involved in a house fire and sustains extensive


partial thickness burns to his torso
and thigh. Two weeks post operatively he develops oedema of both lower
legs. The most likely cause of
this is:
A. Iliofemoral deep vein thrombosis
B. Venous obstruction due to scarring
C. Hypoalbuminaemia
D. Excessive administration of intravenous fluids
E. None of the above
Theme from 2009 Exam

A 45 year old lady has recently undergone a thyroidectomy for


treatment of medullary thyroid cancer.
Which of the following tumour markers is used clinically to screen for
recurrence?
A. Free T3
B. Thyroglobulin
C. Calcitonin
D. Free T4
E. Thyroid stimulating hormone
Theme from 2011 Exam

A 22 year old man is kicked in the head during a rugby match. He is


temporarily concussed, but then
regains consciousness. Half an hour later he develops slurred speech,
ataxia and loses consciousnesses.
On arrival in hospital he is intubated and ventilated. A CT Scan is
performed which shows an extradural
haematoma. What is the most likely cause?
A. Basilar artery laceration
B. Middle meningeal artery laceration
C. Laceration of the sigmoid sinus
D. Laceration of the anterior cerebral artery
E. Laceration of the middle cerebral artery
Theme based on September 2011 Exam
Theme from April 2013 Exam

A baby is born by normal vaginal delivery at 39 weeks gestation.


Initially all appears well and then the
clinical staff become concerned because the baby develops recurrent
episodes of cyanosis. These are
worse during feeding and improve dramatically when the baby cries.
The most likely underlying
diagnosis is:
A. Choanal atresia
B. Oesophageal reflux
C. Tetralogy of Fallot
D. Oesophageal atresia
E. Congenital diaphragmatic hernia
Theme from 2011 exam
Theme from April 2013 Exam

A 20 year old man is involved in a road traffic accident. Following the


incident he is unable to extend
his wrist. However, this improves over the following weeks. Which type
of injury is he most likely to
have sustained?
A. Radial nerve neurotmesis
B. Radial nerve neuropraxia
C. Axillary nerve axonotmesis
D. Ulnar nerve neuropraxia
E. Ulnar nerve axonotmesis
Theme from April 2011 Exam
Transient loss of function makes neuropraxia the most likely injury. The
wrist extensors are innervated
by the radial nerve making this the most likely site of injury.

Theme: Thyroid neoplasms


A. Follicular carcinoma
B. Anaplastic carcinoma
C. Medullary carcinoma
D. Papillary carcinoma
E. Lymphoma
F. Hashimotos thyroiditis
G. Graves disease

For the following histological descriptions please select the most likely
underlying thyroid
neoplasm. Each option may be used once, more than once or not at all.
51. A 22 year old female undergoes a thyroidectomy. The resected
specimen shows a non
encapsulated tumour with papillary projections and pale empty nuclei.
Papillary carcinoma
Theme from April 2012
The presence of papillary structures together with the cytoplasmic
features described is strongly
suggestive of papillary carcinoma. They are seldom encapsulated.
52. A thyroidectomy specimen from a 43 year old lady shows a mass
with prominent oxyphil cells
and scanty thyroid colloid.
You answered Medullary carcinoma
The correct answer is Follicular carcinoma
Hurthle cell tumours are a variant of follicular neoplasms in which
oxyphil cells predominate.
They have a poorer prognosis than conventional follicular neoplasms
53. A 32 year old lady undergoes a thyroidectomy for a mild goitre. The
resected specimen shows an
intense lymphocytic infiltrate with acinar destruction and fibrosis.
Hashimotos thyroiditis
Lymphocytic infiltrates and fibrosis are typically seen in Hashimotos
thyroiditis. In Lymphoma
only dense lymphatic type tissue is usually present.

From which of the following cell types do giant cells most commonly
originate?
A. Neutrophils
B. Myofibroblasts
C. Fibroblasts
D. Macrophages
E. Goblet cells
Theme from September 2011 and 2009 Exam
A 43 year old lady with hypertension is suspected of having a
phaeochromocytoma. Which of the
following investigations is most likely to be beneficial in this situation?
A. Dexamethasone suppression test
B. Urinary 5-Hydroxyindoleacetic Acid (5-HIAA)
C. Histamine provocation test
D. Tyramine provocation test
E. Urinary vanillymandelic acid measurements
Theme from September 2011 Exam
Theme from September 2012 Exam

A 46 year old lady presents with symptoms of diarrhoea, weight loss of


10 Kg and a skin rash of
erythematous blisters involving the abdomen and buttocks. The blisters
have an irregular border and
both intact and ruptured vesicles. What is the most likely diagnosis?
A. Colonic adenocarcinoma
B. Pancreatic adenocarcinoma
C. Tropical sprue
D. Glucagonoma
E. Insulinoma
Theme from September 2011 Exam
Theme from September 2012 Exam
Glucagonoma is strongly associated with necrolytic migratory
erythema.

A 56 year old man presents with symptoms of neuropathic facial pain


and some weakness of the
muscles of facial expression on the right side. On examination he has a
hard mass approximately 6cm
anterior to the right external auditory meatus. What is the most likely
diagnosis?
A. Pleomorphic adenoma
B. Adenocarcinoma
C. Mucoepidermoid carcinoma
D. Adenoid cystic carcinoma
E. Lymphoma
Theme from September 2011 Exam
The patient is most likely to have a malignant lesion within the parotid.
Of the malignancies listed;
adenoid cystic carcinoma has the greatest tendency to perineural
invasion.

A 45 year old women with a thyroid carcinoma undergoes a total


thyroidectomy. The post operative
histology report shows a final diagnosis of medullary type thyroid
cancer. Which of the tests below is
most likely to be of clinical use in screening for disease recurrence?
A. Serum CA 19-9 Levels
B. Serum thyroglobulin levels
C. Serum PTH levels
D. Serum calcitonin levels
E. Serum TSH levels
Theme from September 2012 Exam

A 56 year old man has undergone a radical nephrectomy. The


pathologist bisects the kidney and
identifies a pink fleshy tumour in the renal pelvis. What is the most
likely disease?
A. Renal cell carcinoma
B. Transitional cell carcinoma
C. Angiomyolipoma
D. Phaeochromocytoma
E. Renal adenoma
Most renal tumours are yellow or brown in colour. TCC's are one of the
few tumours to appear pink.
Theme from April 2012

73 year old man presents with haemoptysis and is suspected of suffering


from lung cancer. On
examination he has an enlarged supraclavicular lymph node. Which of
the following features is most
likely to be present on histological examination?
A. Increased mitoses
B. Apoptosis
C. Barr Bodies
D. Multinucleate giant cells
E. Granuloma
Theme from 2011 Exam
Increased mitoses are commonly seen in association with malignant
transformation of cells. Apoptosis is
not a common feature of metastatic cancer. Barr Bodies are formed
during X chromosome inactivation
in female somatic cells.

Which of the following pathological explanations best describes the


initial pathological processes
occurring in an abdominal aortic aneurysm in an otherwise well 65 year
old, hypertensive male?
A. Loss of elastic fibres from the adventitia
B. Loss of collagen from the adventitia
C. Loss of collagen from the media
D. Loss of elastic fibres from the media
E. Decreased matrix metalloproteinases in the adventitia
Theme from April 2012 Exam
Theme from April 2013 Exam

A 28 year old lady has a malignant melanoma removed from her calf.
Which of the following
pathological criteria carries the greatest prognostic weighting?
A. Vascular invasion
B. Abnormal mitoses
C. Breslow thickness
D. Perineural invasion
E. Lymphocytic infiltrates
Theme from April 2012 Exam
A 20 year old girl presents with a thyroid cancer, she is otherwise well
with no significant family
history. On examination she has a nodule in the left lobe of the thyroid
with a small discrete mass
separate from the gland itself. Which of the following is the most likely
cause?
A. Follicular carcinoma
B. Anaplastic carcinoma
C. Medullary carcinoma
D. Papillary carcinoma
E. B Cell Lymphoma
Theme from September 2011 Exam

A 28 year old lady is breast feeding her first child. She presents with
discomfort of the right breast.
Clinical examination demonstrates erythema and an area that is
fluctuant. Aspiration and culture of the
fluid is most likely to demonstrate infection with which of the following
organisms?
A. Clostridium perfringens
B. Staphylococcus aureus
C. Streptococcus pyogenes
D. Staphylococcus epidermidis
E. Actinomycosis
Theme from 2013, 2010 and 2009 Exam
Staphylococcus aureus is the commonest cause. The infants mouth is
usually the source as it damages
the nipple areolar complex allowing entry of bacteria.

Theme: Tumour markers


A. Invasive ductal carcinoma of the breast
B. Prostate cancer
C. Gastric cancer
D. Ovarian cancer
E. Colorectal cancer
F. Pancreatic adenocarcinoma
G. Seminoma testicular cancer
H. Non-seminomatous testicular cancer
I. Hepatocellular carcinoma
For each tumour marker please select the most likely underlying
malignancy. Each option may be used
once, more than once or not at all.
77. Raised beta-human chorionic gonadotropin with a raised alpha-feto
protein level
Non-seminomatous testicular cancer
Theme from April 2012 Exam
A raised alpha-feto protein level excludes a seminoma
78. Elevated CA 19-9
Pancreatic adenocarcinoma
79. Raised alpha-feto protein level in a 54-year-old woman
Hepatocellular carcinoma
Tumour markers
Theme from January 2013 exam

A 56 year old man is diagnosed as having a glioma. From which of the


following cell types do these
tumours usually originate?
A. Astrocytes
B. Oligodendrocytes
C. Ependymal cells
D. Squamous cells
E. Neuroglial cells
Theme from January 2012 Exam
Gliomas originate from glial (otherwise known as neuroglial) cells.
These serve a structural function in
the CNS. The tumours produced may resemble a number of CNS cell
types. Tumours are therefore
named according to the cells they resemble rather than the origin.
Where this is not possible they are
termed gliomas.

A 63 year old man finds that he has to stop walking after 100 yards due
to bilateral calf pain. He finds
that bending forwards and walking up hill helps. He is able to ride a
bike without any pain. What is the
most likely underlying cause?
A. Lumbar canal stenosis
B. Diabetic neuropathy
C. Aorto-iliac occlusion
D. Occlusion of the superficial femoral artery
E. Pelvic rheumatoid arthritis
Theme from April 2012 Exam
Theme from April 2013 Exam
The positional nature of the pain and the fact that improves with
walking uphill makes an underlying
vascular aetiology far less likely.

A 73 year old lady is admitted for a laparoscopic cholecystectomy.


During her pre-operative assessment
it is noted that she is receiving furosemide for the treatment of
hypertension. Approximately what
proportion of the sodium that is filtered at the glomerulus will be
subsequently excreted?
A. Up to 25%
B. Upt to 75%
C. Between 3 and 5%
D. <2%
E. Between 1 and 2%
Theme from 2010 Exam
A 59 year old man presents with recurrent episodes of urinary sepsis. In
his history he mentions that he
has suffered from recurrent attacks of left iliac fossa pain over the past
few months. He has also notices
bubbles in his urine. He undergoes a CT scan which shows a large
inflammatory mass in the left iliac
fossa. No other abnormality is detected. The most likely diagnosis is:
A. Ulcerative colitis
B. Crohns disease
C. Mesenteric ischaemia
D. Diverticular disease
E. Rectal cancer
Theme from 2009 and 2011 Exam

Theme: Lung cancer


A. Adenocarcinoma
B. Small cell lung cancer
C. Large cell lung cancer
D. Squamous cell carcinoma
Please select the most likely lung cancer variant for the scenario
described. Each option may be used
once, more than once or not at all.
107. A 73 year old heavy smoker presents with haemoptysis. On
examination he is cachectic and
shows evidence of clubbing. Imaging shows a main bronchial tumour
with massive mediastinal
lymphadenopathy together with widespread visceral metastases.
Small cell lung cancer
Theme from April 2012
Small cell carcinoma is associated with disseminated disease at
presentation in the majority of
cases. Most cases occur in the main airways and paraneoplastic features
are common.
108. A 68 year old female who has never smoked presents with a mass at
the periphery of her right
lung.
Adenocarcinoma
Adenocarcinomas are the most common tumour type present in never
smokers. They are usually
located at the periphery.
109. An 85 year old man presents with a cough and haemoptysis. He has
a modest smoking history of
15 pack years. He is found to have a tumour located in the right main
bronchus, with no
evidence of metastatic disease. He decides no undergo any treatment
and he remains well for a
further 12 months before developing symptomatic metastasis.
Squamous cell carcinoma
Squamous cell carcinomas are reported to be more slow growing and
are typically centrally
located. Small cell carcinomas are usually centrally located. However,
small cell carcinomas
would seldom be associated with a survival of a year without treatment.

A 63 year old male presents with several episodes of haematuria. He


suffers from COPD secondary to
long term smoking. Which is the most likely underlying cause?
A. Renal cortical adenoma
B. Renal adenocarcinoma
C. Nephroblastoma
D. Transitional cell carcinoma of the bladder
E. Adenocarcinoma of the bladder
Theme from 2009 Exam
TCC is the most common subtype and is strongly linked to smoking.
The important point to note in this
question is the term most likely as renal adenocarcinoma may produce
similar symptoms but is less
likely.
A male infant is born prematurely at 34 weeks gestation by emergency
cesarean section. He initially
appears to be stable. However, over the ensuing 24 hours he develops
worsening neurological function.
Which of the following processes is most likely to have occurred?
A. Extra dural haemorrhage
B. Sub dural haemorrhage
C. Sub arachnoid haemorrhage
D. Intraventricular haemorrhage
E. Arteriovenous malformation
Theme from April 2012 Exam

A 43 year old man presents with dyspepsia and undergoes an upper GI


endoscopy. During the procedure
diffuse gastric and duodenal ulcers are identified. A Clo test confirms
the presence of Helicobacter
pylori infection. What is the most likely explanation for the ulcers?
A. Decreased gastric motility
B. Increased urease activity
C. Decreased release of mucous and bicarbonate
D. Decreased gastrin levels
E. Increased acid production
Theme from April 2011 Exam
H-Pylori has a number of pathological effects. In this question the main
issue is by what mechanism the
organism is able to induce both gastric and duodenal ulceration.
Without modestly elevated acid levels,
the duodenum would not undergo gastric metaplasia. H-Pylori cannot
colonise duodenal mucosa and
therefore the development of ulcers at this site can only occur in those
who have undergone metaplastic
transformation (mediated by increased acidity).

A 25 year old male pedestrian is involved in a road traffic accident. He


sustains multiple injuries and is
admitted to the intensive care unit, intubated and ventilated. Over the
next week he develops adult
respiratory distress syndrome. What is the main reason for hypoxaemia
in this condition?
A. Increased lung compliance
B. Reduced diffusion
C. Reduced surfactant
D. Reduced elastase
E. Left to right shunt
Theme from 2011 Exam
Theme from January 2013 Exam
The diffuse lung injury, which is associated with loss of surfactant and
increased elastase release from
neutrophils, results in fluid accumulation. This leads to reduced
diffusion, which is the main reason for
hypoxaemia.

A 48 year old women presents with recurrent loin pain and fevers.
Investigation reveals a staghorn
calculus of the left kidney. Infection with which of the following
organisms is most likely?
A. Staphylococcus saprophyticus
B. Proteus mirabilis
C. Klebsiella
D. E-Coli
E. Staphylococcus epidermidis
Theme from April 2012 Exam
Infection with Proteus mirabilis accounts for 90% of all proteus
infections. It has a urease producing
enzyme. This will tend to favor urinary alkalinisation which is a relative
per-requisite for the formation
of staghorn calculi.

Theme: Facial nerve palsy


A. Adenoid cystic carcinoma
B. Cerebrovascular accident
C. Petrous temporal fracture
D. Warthins tumour
E. Sarcoidosis
F. Pleomorphic adenoma
G. Cholesteatoma
Please select the most likely cause of facial nerve palsy for the scenario
given. Each option may be used
once, more than once or not at all.
3. A 22 year old man presents with symptoms of lethargy and bilateral
facial nerve palsy. On
examination he has bilateral parotid gland enlargement.
You answered Adenoid cystic carcinoma
The correct answer is Sarcoidosis
Theme from 2011 Exam
Facial nerve palsy is the commonest neurological manifestation of
sarcoid. It usually resolves.
The absence of ear discharge or discrete lesion on palpation is against
the other causes.
4. A 21 year old man presents with a unilateral facial nerve palsy after
being hit in the head. On
examination he has a right sided facial nerve palsy and a watery
discharge from his nose.
Petrous temporal fracture
Nasal discharge of clear fluid and recent head injury makes a basal
skull fracture the most
likely underlying diagnosis.
5. A 43 year old lady presents with symptoms of chronic ear discharge
and a right sided facial
nerve palsy. On examination she has foul smelling fluid draining from
her right ear and a
complete right sided facial nerve palsy.
Cholesteatoma
Foul smelling ear discharge and facial nerve weakness is likely to be due
to cholesteatoma. The
presence of a neurological deficit is a sinister feature.
In patients with an annular pancreas where is the most likely site of
obstruction?
A. The first part of the duodenum
B. The second part of the duodenum
C. The fourth part of the duodenum
D. The third part of the duodenum
E. The duodeno-jejunal flexure
Theme from 2011 Exam
Theme from September 2012 Exam

A 53 year old man presents with dyspepsia. An upper GI endoscopy is


performed and Helicobacter
pylori is identified. A duodenal ulcer is present in the first part of the
duodenum. Duodenal biopsies are
taken and demonstrate epithelium that resembles cells of the gastric
antrum. Which of the following is
the most likely explanation for this process?
A. Hyperplasia of the crypts of Lieberkhun
B. Duodenal metaplasia
C. Duodenal dysplasia
D. Duodenal carcinoma
E. Hyptertrophy of Brunners glands
Metaplasia = cell type
conversion
Theme in January 2012 exam

A 73 year old man is recovering following an emergency Hartmans


procedure performed for an
obstructing sigmoid cancer. The pathology report shows a moderately
differentiated adenocarcinoma
that invades the muscularis propria, 3 of 15 lymph nodes are involved
with metastatic disease. What is
the correct stage for this?
A. Astler Coller Stage B2
B. Dukes stage A
C. Dukes stage B
D. Dukes stage C
E. Dukes stage D
Theme from September 2011 Exam

A 56 year old man is diagnosed with an abdominal aortic aneurysm and


undergoes a CT scan to asses
the size of the aorta. During the course of his investigations a lesion of
the adrenal gland is identified. It
measures 1.5 cm in diameter and the gland is otherwise normal. What is
the most likely diagnosis?
A. Adrenal gland metastasis
B. Adrenal gland arterio-venous malformation
C. Adrenal cyst
D. Phaeochromocytoma
E. Adrenal cortical adenoma
25% of all adrenal lesions >4cm in
diameter are malignant
Theme from 2011 Exam
Incidentalomas of the adrenal gland are common and represent the
most likely lesion in this scenario.
Clearly the other lesions are all possibilities but are unlikely.

Theme: Chest pain


A. Pulmonary embolism
B. Acute exacerbation asthma
C. Physiological
D. Mitral valve stenosis
E. Aortic dissection
F. Mitral regurgitation
G. Bronchopneumonia
H. Tuberculosis
I. None of the above
What is the most likely diagnosis for the scenario given? Each option
may be used once, more than once
or not at all.
47. A 28 year old Indian woman, who is 18 weeks pregnant, presents
with increasing shortness of
breath, chest pain and coughing clear sputum. She is apyrexial, blood
pressure is 140/80
mmHg, heart rate 130 bpm and saturations 94% on 15L oxygen. On
examination there is a
mid diastolic murmur, there are bibasal crepitations and mild pedal
oedema. She suddenly
deteriorates and has a respiratory arrest. Her chest x-ray shows a
whiteout of both of her
lungs.
You answered Tuberculosis
The correct answer is Mitral valve stenosis
Mitral stenosis is the commonest cause of cardiac abnormality
occurring in pregnant women.
Mitral stenosis is becoming less common in the UK population, however
should be
considered in women from countries were there is a higher incidence of
rheumatic heart
disease. Mitral stenosis causes a mid diastolic murmur which may be
difficult to auscultate
unless the patient is placed into the left lateral position. These patients
are at risk of atrial
fibrillation (up tp 40%), which can also contribute to rapid
decompensation. Physiological
changes in pregnancy may cause an otherwise asymptomatic patient to
suddenly deteriorate.
Balloon valvuloplasty is the treatment of choice.
48. A 28 year old woman, who is 30 weeks pregnant, presents with
sudden onset chest pain
associated with loss of consciousness. Her blood pressure is 170/90
mmHg, saturations on
15L oxygen 93%, heart rate 120 bpm and she is apyrexial. On
examination there is an early
diastolic murmur, occasional bibasal creptitations and mild peal
oedema. An ECG shows ST
elevation in leads II, III and aVF.
Aortic dissection
Aortic dissection is associated with the 3rd trimester of pregnancy,
connective tissue disorders
(Marfan's, Ehlers- Danlos) and bicuspid valve. Patients may complain
of a tearing chest pain
or syncope. Clinically they may be hypertensive. The right coronary
artery may become
involved in the dissection, causing myocardial infarct in up to 2% cases
(hence ST elevation
in the inferior leads). An aortic regurgitant murmur may be
auscultated.
49. A 28 year old woman, who is 18 weeks pregnant, presents with
sudden chest pain. Her blood
pressure is 150/70 mmHg, saturations are 92% on 15L oxygen and her
heart rate is 130 bpm.
There are no murmurs and her chest is clear. There is signs of
thrombophlebitis in the left leg.
Pulmonary embolism
Chest pain, hypoxia and clear chest on auscultation in pregnancy should
lead to a high
suspicion of pulmonary embolism.
Theme question in September 2011 exam
Pregnant women can decompensate rapidly from
cardiac compromise.

A 63 year old man has a history of claudication that has been present for
many years. He is recently
evaluated in the clinic and a duplex scan shows that he has an 85%
stenosis of the superficial femoral
artery. Two weeks later he presents with a 1 hour history of severe pain
in his leg. On examination he
has absent pulses in the affected limb and it is much cooler than the
contra-lateral limb. Which process
best accounts for this presentation?
A. Thrombosis
B. Embolus
C. Atheroma growth
D. Sub intimal dissection
E. Anaemia
Theme from April 2012 Exam

Theme: Paediatric ano-rectal disorders


A. Ulcerative colitis
B. Juvenile polyps
C. Haemorroids
D. Intussceception
E. Rectal cancer
F. Anal fissure
G. Arteriovenous malformation
Please select the most likely cause for the condition described. Each
option may be used once, more than
once or not at all.
79. A 4 year old boy is brought to the clinic. He gives a history of
difficult, painful defecation
with bright red rectal bleeding.
Anal fissure
Theme from April 2012 Exam
Painful rectal bleeding in this age group is typically due to a fissure.
Treatment should include
stool softeners and lifestyle advice.
80. A 2 year old has a history of rectal bleeding. The parents notice that
post defecation, a cherry
red lesion is present at the anal verge.
You answered Arteriovenous malformation
The correct answer is Juvenile polyps
Theme from September 2012 Exam
These lesions are usually hamartomas and this accounts for the colour
of the lesions. Although
the lesions are not themselves malignant they serve as a marker of an
underlying polyposis
disorder.
81. A 12 year old is brought to the colorectal clinic with a history of
rectal bleeding, altered bowel
habit, weight loss and malaise. Abdominal examination is normal.
Ulcerative colitis
The systemic features in the history are strongly suggestive of
inflammatory bowel disease
rather than the other causes.

Which of the following processes facilitates phagocytosis?


A. Apoptosis
B. Opsonisation
C. Proteolysis
D. Angiogenesis
E. Necrosis
Theme from 2008 Exam

A 55 year old man undergoes a colonoscopy and a colonic polyp is


identified. It has a lobular
appearance and is located on a stalk in the sigmoid colon. Which of the
processes below best accounts
for this disease?
A. Apoptosis
B. Metaplasia
C. Dysplasia
D. Calcification
E. Degeneration
Theme from April 2012 Exam
Most colonic polyps described above are adenomas. These may have
associated dysplasia. The more
high grade the dysplasia the greater the level of clinical concern.
A 70 year old male presents with painless frank haematuria. Clinical
examination is unremarkable.
Routine blood tests reveal a haemoglobin of 18g/dl but are otherwise
normal. What is the most likely
underlying diagnosis?
A. Squamous cell carcinoma of the bladder
B. Adenocarcinoma of the prostate
C. Adenocarcinoma of the kidney
D. Wilms tumour
E. Transitional cell carcinoma of the renal pelvis
Theme from April 2012 Exam
Polycythaemia is a recognised feature of renal cell carcinoma. Wilms
tumours most commonly occur in
children.

A newborn infant is noted to have a unilateral cleft lip only. What is the
most likely explanation for this
process?
A. Incomplete fusion of the second branchial arch
B. Incomplete fusion of the nasolabial muscle rings
C. Incomplete fusion of the first branchial arch
D. Incomplete fusion of the third branchial arch
E. Incomplete fusion of the secondary palate
Theme from April 2012 Exam
Unilateral isolated cleft lip represents a failure of nasolabial ring fusion.
It is not related to branchial
arch fusion. Arch disorders have a far more profound phenotype and
malformation sequences.

Theme: Pharyngitis
A. Infectious mononucleosis
B. Acute bacterial tonsillitis
C. Quinsy
D. Lymphoma
E. Diptheria
Please select the most likely underlying cause for the following patients
presenting with pharyngitis.
Each option may be used once, more than once or not at all.
4. An 8 year old child presents with enlarged tonsils that meet in the
midline and are covered with
a white film that bleeds when you attempt to remove it. He is pyrexial
but otherwise well.
You answered Diptheria
The correct answer is Acute bacterial tonsillitis
Theme from April 2012 Exam
In acute tonsillitis the tonsils will often meet in the midline and may be
covered with a
membrane. Individuals who are systemically well are unlikely to have
diptheria.
5. A 10 year old child presents with enlarged tonsils that meet in the
midline. Oropharyngeal
examination confirms this finding and you also notice peticheal
haemorrhages affecting the
oropharynx. On systemic examination he is noted to have splenomegaly.
You answered Lymphoma
The correct answer is Infectious mononucleosis
A combination of pharyngitis and tonsillitis is often seen in glandular
fever. Antibiotics
containing penicillin may produce a rash when given in this situation,
leading to a mistaken
label of allergy.
6. A 19 year old man has had a sore throat for the past 5 days. Over the
past 24 hours he has
notices increasing and severe throbbing pain in the region of his right
tonsil. He is pyrexial and
on examination he is noted to have a swelling of this area.
Quinsy
Unilateral swelling and fever is usually indicative of quinsy. Surgical
drainage usually
produces prompt resolution of symptoms.
Which of the following tumours are most likely to give rise to para-
aortic nodal metastasis early?
A. Ovarian
B. Bladder
C. Rectal
D. Caecal
E. Cervical
Theme from 2009 Exam
Ovarian tumours are supplied by the ovarian vessels, these branch
directly from the aorta. The cervix
drains to the internal and external iliac nodes.

Theme: Thyroid disorders


A. Sick euthyroid
B. Hyperthyroidism
C. Hypothyroidism
D. Normal euthyroid
E. Anxiety state
F. Factitious hyperthyroidism
For each of the scenarios please match the scenario with the most likely
underlying diagnosis. Each
answer may be used once, more than once or not at all.
10. A 33 year old man is recovering following a protracted stay on the
intensive care unit recovering
from an anastomotic leak following a difficult Trans hiatal
oesophagectomy. His progress is slow,
and the intensive care doctors receive the following thyroid function test
results:
TSH 1.0 u/L
Free T4 8
T3 1.0 (1.2-3.1 normal)
Sick euthyroid
Theme from April 2012 Exam
Sick euthyroid syndrome is caused by systemic illness. With this, the
patient may have an
apparently low total and free T4 and T3, with a normal or low TSH.
Note that the levels are only
mildly below normal.
11. A 28 year old female presents to the general practitioner with
symptoms of fever and diarrhoea.
As part of her diagnostic evaluation the following thyroid function tests
are obtained:
TSH < 0.01
Free T4 30
T3 4.0
Hyperthyroidism
The symptoms are suggestive of hyperthyroidism. This is supported by
the abnormal blood
results; suppressed TSH with an elevated T3 and T4.
12. A 19 year old lady presents with palpitations. The medical officer
takes a blood sample for
thyroid function tests. The following results are obtained:
TSH > 6.0
Free T4 20
T3 2.0
You answered Factitious hyperthyroidism
The correct answer is Hypothyroidism
An elevated TSH with normal T4 indicates partial thyroid failure. This
is caused by Hashimotos,
drugs (lithium, antithyroids) and dyshormogenesis.

Theme: Cardiac murmurs


A. Pulmonary stenosis
B. Mitral regurgitation
C. Tricuspid regurgitation
D. Aortic stenosis
E. Mitral stenosis
F. Aortic sclerosis
What is the most likely cause of the cardiac murmur in the following
patients? Each option may be used
once, more than once or not at all.
18. A 35 year old Singaporean female attends a varicose vein pre
operative clinic. On auscultation
a mid diastolic murmur is noted at the apex. The murmur is enhanced
when the patient lies in
the left lateral position.
Mitral stenosis
Theme from September 2011 exam
A mid diastolic murmur at the apex is a classical description of a mitral
stenosis murmur. The
most common cause is rheumatic heart disease. Complications of mitral
stenosis include atrial
fibrillation, stroke, myocardial infarction and infective endocarditis.
19. A 22 year old intravenous drug user is found to have a femoral
abscess. The nursing staff
contact the on call doctor as the patient has a temperature of 39oC. He
is found to have a pan
systolic murmur loudest at the left sternal edge at the 4th intercostal
space.
Tricuspid regurgitation
Intravenous drug users are at high risk of right sided cardiac valvular
endocarditis. The
character of the murmur fits with a diagnosis of tricuspid valve
endocarditis.
20. An 83 year old woman is admitted with a left intertrochanteric neck
of femur fracture. On
examination the patient is found to have an ejection systolic murmur
loudest in the aortic
region. There is no radiation of the murmur to the carotid arteries. Her
ECG is normal.
You answered Aortic stenosis
The correct answer is Aortic sclerosis
The most likely diagnosis is aortic sclerosis. The main differential
diagnosis is of aortic
stenosis, however as there is no radiation of the murmur to the carotids
and the ECG is
normal, this is less likely.

A 33 year old man is involved in a road traffic accident. He is initially


stable and transferred to the
accident and emergency department. On arrival he is catheterised. One
minute later he becomes
hypotensive, with evidence of angioedema surrounding his penis. What
is the most likely explanation
for this event?
A. Type V latex hypersensitivity reaction
B. Type IV latex hypersensitivity reaction
C. Type III latex hypersensitivity reaction
D. Type I latex hypersensitivity reaction
E. Type II latex hypersensitivity reaction
Theme from January 2013 exam
Theme from April 2012 Exam

A 52 year old man with dyspepsia is found to have a duodenal ulcer. A


CLO test is taken and is positive.
Which statement relating to the likely causative organism is false?
A. It is a gram negative organism
B. It lives only on gastric type mucosa
C. It may occupy areas of ectopic gastric metaplasia
D. In patients who are colonised there is commonly evidence of fundal
gastritis on
endoscopy
E. It produces a powerful urease that forms the basis of the Clo test
Helicobacter pylori accounts for >75% cases of duodenal ulceration. It
may be diagnosed with either
serology, microbiology, histology or CLO testing.
Theme from January 2011 Exam
Helicobacter pylori rarely produces any typical features on endoscopy.
Where infection is suspected the
easiest course of action is to take an antral biopsy for Clo testing in the
endoscopy suite.

A 13 month old boy is brought to the surgical clinic by his mother


because his left testicle is not located
in the scrotum. At which of the following sites would the testicle be
located if it were an ectopic testis?
A. Canalicular
B. Inguinal
C. External inguinal ring
D. Superficial inguinal pouch
E. High scrotal
Theme from September 2011 Exam
Theme from January 2012 Exam
Ectopic testes are those that come to lie outside the normal range of
embryological descent (i.e. in the
superficial inguinal pouch). Other sites of ectopic testes include; base of
penis, femoral and perineal.

A splenectomy increases the risk of infection from all the following


organisms except?
A. Pneumococcus
B. Klebsiella
C. Haemophilus influenzae
D. Staphylococcus aureus
E. Neisseria meningitidis
Theme from 2010 Exam

An 18 month old boy presents with recurrent urinary tract infections.


As part of the diagnostic work-up
he is noted to have abnormal renal function. An ultrasound scan is
performed and shows bilateral
hydronephrosis. What is the most likely underlying diagnosis?
A. Urethral valves
B. Meatal stenosis
C. Hydronephrosis
D. Pelvico-ureteric junction obstruction
E. Benign prostatic hyperplasia
Theme from April 2012 Exam
A posterior urethral valve is an obstructive, developmental uropathy
that usually affects male infants
(incidence 1 in 8000). Diagnostic features include bladder wall
hypertrophy, hydronephrosis and bladder
diverticula.

Theme: Causes of chest pain


A. Pulmonary embolism
B. Anterior myocardial infarction
C. Inferior myocardial infarction
D. Proximal aortic dissection
E. Distal aortic dissection
F. Boerhaave Syndrome
G. Mallory weiss tear
H. Perforated gastric ulcer
Please select the most likely cause of chest pain for the scenario given.
Each option may be used once,
more than once or not at all.
67. A 52 year old male presents with tearing central chest pain. On
examination he has an aortic
regurgitation murmur. An ECG shows ST elevation in leads II, III and
aVF.
You answered Inferior myocardial infarction
The correct answer is Proximal aortic dissection
Theme from 2011 Exam
An inferior myocardial infarction and AR murmur should raise
suspicions of an ascending
aorta dissection rather than an inferior myocardial infarction alone.
Also the history is more
suggestive of a dissection. Other features may include pericardial
effusion, carotid dissection
and absent subclavian pulse.
68. A 52 year old male presents with central chest pain and vomiting. He
has drunk a bottle of
vodka. On examination there is some mild crepitus in the epigastric
region.
You answered Perforated gastric ulcer
The correct answer is Boerhaave Syndrome
The Mackler triad for Boerhaave syndrome: vomiting, thoracic pain,
subcutaneous
emphysema. It commonly presents in middle aged men with a
background of alcohol abuse.
69. A 52 year old male presents with central chest pain. On examination
he has an mitral
regurgitation murmur. An ECG shows ST elevation in leads V1 to V6.
There is no ST
elevation in leads II, III and aVF.
Anterior myocardial infarction
The most likely diagnosis is an anterior MI. As there are no ST changes
in the inferior leads,
aortic dissection is less likely.

A 78 year old lady presents with a tender swelling in her right groin. On
examination there is a tender
swelling that lies lateral to the pubic tubercle. It has a cough impulse.
What is the most likely underlying
diagnosis?
A. Thrombophlebitis of the great saphenous vein
B. Femoral hernia
C. Thrombophlebitis of saphena varix
D. Inguinal hernia
E. Obturator hernia
Theme from April 2012 Exam
Whilst a thrombophlebitis of a saphena varix may cause a tender
swelling at this site, it would not
usually be associated with a cough impulse.

A 22 year old man undergoes a splenectomy for an iatrogenic splenic


injury. On the second post
operative day a full blood count is performed. Which of the following
components of the full blood
count is the first to be affected ?
A. Erythrocyte count
B. Reticulocyte count
C. Eosinophil count
D. Monocyte count
E. Lymphocyte count
Theme from January 2012 Exam
The granulocyte and platelet count are the first to be affected following
splenectomy. Then reticulocytes
increase. Although a lymphocytosis and monocytosis are reported, these
take several weeks to develop.

S-ar putea să vă placă și